Tìm MVUE độc đáo


10

Câu hỏi này là từ Giới thiệu về Thống kê toán học Phiên bản thứ 6 của Robert Hogg, vấn đề 7.4.9 ở trang 38.

Đặt là iid với pdf zero ở nơi khác, trong đó .X1,...,Xnf(x;θ)=1/3θ,θ<x<2θ,θ>0

(a) Tìm mle củaθ^θ

(b) có đủ số liệu thống kê cho không? Tại sao ?θ^θ

(c) Có phải là MVUE duy nhất của ? Tại sao ?(n+1)θ^/nθ

Tôi nghĩ rằng tôi có thể giải (a) và (b), nhưng tôi bị nhầm lẫn bởi (c).

Cho một):

Đặt là thống kê đơn hàng.Y1<Y2<...Yn

L(θ;x)=13θ×13θ×...×13θ=1(3θ)n khi và ; ở nơi khácθ<y1yn<2θL(θ;x)=0

dL(θ;x)dθ=n(3θ)n1 , vì , chúng ta có thể thấy đạo hàm này là âm,θ>0

vì vậy khả năng hàm đang giảm.L(θ;x)

Từ và , và y n < 2 θ ) ( θ > - y 1 θ > y n / 2 ) , θ > m một x ( - y 1 , y n / 2 )(θ<y1yn<2θ) (θ>y1θ>yn/2),θ>max(y1,yn/2)

θ θ > m a x ( - y 1 , y n / 2 ) θ = m a x ( - y 1 , y n / 2 )L(θ,x) đang giảm, vì vậy khi có giá trị mẫu nhất, hàm khả năng sẽ đạt được tối đa, vì , khi , hàm khả năng sẽ đạt được giá trị tối đa.θθ>max(y1,yn/2)θ=max(y1,yn/2)

q = m một x ( - y 1 , y n / 2 ) mleθ^=max(y1,yn/2)

Dành cho (b):

f(x1;θ)f(x2;θ)...f(xn;θ)=1(3θ)ninI(θ<xi<2θ)=1(3θ)nI(max(xi)<2θ)×1

y n = m a x ( x i ) θ y n / 2 Theo định lý nhân tố của Neyman, là một thống kê đủ cho . Do đó, cũng là một statisitc đầy đủyn=max(xi)θyn/2

Samely

f(x1;θ)f(x2;θ)...f(xn;θ)=1(3θ)ninI(θ<xi<2θ)=1(3θ)nI(min(xi)>θ)×1

y 1 = m i n ( x i ) θ - y 1 Theo định lý nhân tố của Neyman, là một thống kê đủ cho . Do đó, cũng là một statisitc đầy đủ.y1=min(xi)θy1

Đối với (c):

Đầu tiên, chúng tôi tìm thấy CDF củaX

F(x)=θx13θdt=x+θ3θ,θ<x<2θ

Tiếp theo, chúng ta có thể tìm thấy pdf cho cả và từ công thức của cuốn sách để thống kê đơn hàng.Y nY1Yn

f(y1)=n!(11)!(n1)![F(y1)]11[1F(y1)]n1f(y1)=n[1y1+θ3θ]n113θ=n1(3θ)n(2θy1)n1

Samely

f(yn)= =n(yn+θ3θ)n-113θ= =n1(3θ)n(yn+θ)n-1

Tiếp theo, chúng tôi cho thấy sự hoàn chỉnh của họ pdf cho vàf ( y n )f(y1)f(yn)

FTCu(θ)=0θ>0E[bạn(Y1)]= =-θ2θbạn(y1)n1(3θ)n(2θ-y1)n-1dy1= =0-θ2θbạn(y1)(2θ-y1)dy1= =0 . Bằng (dẫn xuất tích phân), chúng ta có thể hiển thị cho tất cả .FTCbạn(θ)= =0θ>0

Do đó, họ pdf đã hoàn tất ..Y1

Samely, vẫn bằng , chúng tôi có thể chỉ ra rằng họ pdf đã hoàn tất.Y nFTCYn

Vấn đề là bây giờ chúng ta cần chỉ ra rằng không thiên vị.(n+1)θ^n

Khiθ^= =-y1

E(y1)=θ2θ(y1)n(3θ)n(2θy1)n1dy1=1(3θ)nθ2θy1d(2θy1)n

Chúng ta có thể giải tích phân bằng cách tích hợp bởi các phần

E(y1)=1(3θ)n[y1(2θy1)nθ2θθ2θ(2θy1)ndy1]=1(3θ)n[θ(3θ)n(3θ)n+1n+1]=θ3θn+1=(n2)θn+1

E((n+1)θ^n)=n+1nE(y1)=n+1n(n2)θn+1=n2nθ

Do đó, không phải là công cụ ước tính không thiên vị của khi q q =-y1(n+1)θ^nθθ^=y1

Khiθ^=yn/2

E(Yn)=θ2θynn(3θ)n(yn+θ)n1dyn=1(3θ)nθ2θynd(yn+θ)n=1(3θ)n[yn(yn+θ)nθ2θθ2θ(yn+θ)ndyn]=1(3θ)n[2θ(3θ)(3θ)n+1n+1]=2θ3θn+1=2n1n+1θ

E((n+1)θ^n)=n+1nE(Yn/2)=n+12nE(Yn)=n+12n2n-1n+1θ= =2n-12nθ

Tuy nhiên, không phải là công cụ ước tính không thiên vị của khi(n+1)θ^nq = y n / 2θθ^= =yn/2

Nhưng câu trả lời của cuốn sách là là một MVUE độc đáo. Tôi không hiểu tại sao nó là MVUE nếu nó là một công cụ ước tính thiên vị.(n+1)θ^n

Hoặc những sai sót của tôi là sai, xin hãy giúp tôi tìm ra những sai lầm, tôi có thể cung cấp cho bạn các tính toán chi tiết hơn.

Cảm ơn rât nhiều.


Tôi không thấy bất kỳ tính toán nào về phân phối của . θ^
whuber

Cảm ơn, người đánh bóng, . Đó là hoặc tùy thuộc vào cái nào lớn hơn. Tôi đã tính toán các bản phân phối cho cả và . Bạn có thể thấy và trong văn bản. -y1yn/2y1ynf(y1)=n1θ^=max(y1,yn/2)y1yn/2y1ynf(yn)=n1f(y1)=n1(3θ)n(2θy1)n1f(yn)=n1(3θ)n(yn+θ)n1
Miền Bắc sâu

Và từ hai bản phân phối ở trên, tôi đã tính và rồiE ( θ ) = E ( Y n / 2 ) E ( n + 1E(θ^)=E(Y1)E(θ^)=E(Yn/2)E(n+1nθ^)
Miền Bắc sâu

Câu trả lời:


6

Làm việc với extrema đòi hỏi sự chăm sóc, nhưng nó không phải là khó khăn. Câu hỏi quan trọng, được tìm thấy gần giữa bài, là

... chúng ta cần chỉ ra rằng là không thiên vị.n+1nθ^n

Trước đó bạn có được

θ^= =tối đa(-y1,yn/2)= =tối đa{-tối thiểu{yTôi},tối đa{yTôi}/2}.

Mặc dù rằng có vẻ lộn xộn, các tính toán trở thành tiểu khi bạn xem xét các tích lũy hàm phân bố . Để bắt đầu với điều này, lưu ý rằng . Đặt là một số trong phạm vi này. Theo định nghĩa,0 qq tF0θ^θt

F(t)= =Pr(θ^t)= =Pr(-y1<t và yn/2t)= =Pr(-ty1y2yn2t).

Đây là cơ hội mà tất cả giá trị nằm giữa và . Những giá trị đó ràng buộc một khoảng dài . Vì phân phối là đồng nhất, nên xác suất mà bất kỳ cụ thể nào nằm trong khoảng này đều tỷ lệ thuận với độ dài của nó:- t 2 t 3 t y in-t2t3tyTôi

Pr(yTôi[-t,2t])= =3t3θ= =tθ.

Vì độc lập, các xác suất này nhân lên, choyTôi

F(t)= =(tθ)n.

Có thể tìm thấy sự mong đợi ngay lập tức bằng cách tích hợp chức năng sống sót trong khoảng thời gian của các giá trị có thể cho , , sử dụng cho biến:θ [ 0 , θ ] y = t / θ1-Fθ^[0,θ]y= =t/θ

E(θ^)= =0θ(1-(tθ)n)dt= =01(1-yn)θdy= =nn+1θ.

(Công thức cho kỳ vọng này được lấy từ tích phân thông thường thông qua tích hợp bởi các bộ phận. Chi tiết được đưa ra ở cuối /stats//a/105464 .)

Rescaling bởi cho(n+1)/n

E(n+1nθ^)= =θ,

QED .


Có một lỗi đánh máy cho công thức cuối cùng, nó phải là không q nθ^θ^n
Deep North

@Deep Oh, tất nhiên! Cảm ơn bạn đã chỉ ra rằng. Bây giờ nó đã được sửa.
whuber
Khi sử dụng trang web của chúng tôi, bạn xác nhận rằng bạn đã đọc và hiểu Chính sách cookieChính sách bảo mật của chúng tôi.
Licensed under cc by-sa 3.0 with attribution required.